Existence de mesures diffuses — Les-mathematiques.net The most powerful custom community solution in the world

Existence de mesures diffuses

Bonjour à tous,

mon message du 1er mai ayant eu peu d'écho, je repose ma question
sous une autre forme dans le PDF joint mais je ne trouve plus le bouton
pour joindre un fichier !

Cordialement,

DS

[ Edit : Ancien titre: existence de mesures diffuses bis
Les deux discussions ont été fusionnées, l'ouverture d'un nouveau sujet n'étant pas utile. ]

Réponses

  • Bonjour à tous,

    voici une question que Gustave Choquet posait dans une épreuve d'examen en 1959 :

    Montrer que sur tout espace compact parfait (sans point isolé)
    il existe des mesures positives diffuses non nulles.
    Indication : on pourra construire une telle mesure comme limite
    d'une suite de mesures discrètes Mn telles que Mn consiste en
    2^n masses ponctuelles égales.

    Voyez-vous comment faire ?

    DS
  • Une stratégie ?

    Je pars d'un point $x$ auquel est associé à un voisinage compact $V_x$ . Choisir $x'$ et $x''$ dans l'intérieur de $V_x$ et leur associer deux voisinages compacts $V_{x'}$ et $V_{x''}$ disjoints et inclus dans $V_x$. Si on a une masse $2^{-n}\delta_x$ on lui associe $2^{-n-1}\delta_{x'} + 2^{-n-1}\delta_{x''}$ à l'étape suivante. Je ne sais pas si ça marche très bien mais ce serait mon idée de départ. La stratégie est de pouvoir contrôler le poids que la suite de mesure met dans un bon voisinage compact d'un point quelconque.

    Je ne sais pas si je suis clair...
  • Noter que puisque l'espace ambient (nommons le $K$) est compact, l'ensemble de ses mesures n'est autre que le dual de $(C^0(K,\R), || \cdot ||_{\infty})$ et l'ensemble de ses mesures de probabilité est compact pour la topologie faible. Donc on pourra construire une suite de mesures à support fini et prendre une valeur d'adhérence.
    Une fonction est un ensemble $f$ de couples tel que pour tous $x,y,z$, si $(x,y)\in f$ et $(x,z)\in f$ alors $y = z$.
  • Le décor est bien planté, merci Foys, mais quelle
    suite de mesures à support fini prendre ?
  • Celles que j'ai proposées ne fonctionnent pas ?
  • D'un autre côté on t'a proposé des pistes et tu n'as rien dit en retour... Si tu ne veux pas des pistes mais des réponses entièrement rédigées précise-le tout de suite...
  • comment joindre un fichier ?
  • Salut Daniel,

    L'attachement de pièces jointes fait partie des fonctionnalités du forum qui sont indisponibles depuis l'attaque pirate ayant eu lieu récemment (avec la messagerie privée, les avatars...).
  • Donc j'insère mon PDF :

    Soit (K,d) un espace métrique compact.
    Soit Fn une suite de parties finies de K.
    Soit Mn la mesure uniforme portée par Fn.
    Soit Ln l’intégrale issue de Mn.

    Chaque Ln est de norme < 1, donc il existe une sous-suite
    Lni qui converge vers une L.
    Comme L est positive, le th de Riesz affirme qu’il existe une mesure borélienne M positive sur K
    telle que L(f) soit l'intégrale de f sur K selon dM.

    Ma question : M est-elle diffuse avec un bon choix des parties Fn ?
  • Tu ne veux vraiment pas tenter la piste proposée ? Elle te semble idiote pour une raison que je ne vois pas ?
  • J'ai l'impression que dianelsaada ne voit pas H.
  • J'ai effectivement un peu de mal à comprendre son attitude. Je comprendrais qu'il trouve ma piste stupide (je n'ai pas poussé l'argument pour vérifier que cela marchait). Je comprendrais qu'il la trouve trop elliptique et demande des détails (elle est effectivement très elliptique, j'ai tout de même été plus loin dans mon raisonnement !). Mais ignorer totalement mes indications (comme il l'a fait sur un autre fil) est une attitude assez étrange. Je ne sais pas s'il a la volonté de m'être désagréable (je ne pense pas l'avoir un jour offensé) ou s'il a simplement une perception différentes des échanges sur un forum.
  • je ne vois pas ce que je ne comprends pas
  • C'est nous qui ne te comprenons pas. H te parle et tu l'ignores totalement, ça ressemble vraiment à du mépris.
  • @Daniel Saada : veux-tu dire que si tu ne réagis pas à mes indications c'est parce que tu ne les comprends pas ?

    Si ce n'est pas le cas peux-tu préciser ta pensée ? Dans la suite, je suppose que j'ai compris ton message. Je trouve ton attitude très irrespecteuse : une personne prend du temps à réfléchir à ton problème et essaye d'engager la conversation avec toi ; de ton côté, tu l'ignores ! Dans la réalité, quand quelqu'un te fait une réponse que tu ne comprends pas, ignores-tu également totalement ton interlocuteur ? Je trouve également ton attitude assez étrange : pourquoi ne pas demander des détails si une question te turlupine et que tu as en face de toi quelqu'un qui semble avoir des idées ? Mais je suis par contre content d'apprendre que tu ne me reproches rien, c'est surtout cela que je voulais savoir.
  • je ne te reproche rien, contrairement à toi...
  • Ça devient assez surréaliste... J'aimerais bien avoir l'avis de quelques personnes de passage sur cette discussion.
  • Question intéressante; je suis d'accord avec l'idée de H.
    Après, il faudrait que je révise ma topologie pour remplir les blancs (j'ai trop l'habitude des espaces métriques).
    Je ne comprends pas bien que Daniel ne réponde pas à H, même pour dire qu'il ne veut pas encore tester sa solution et chercher par lui-même .
  • C'est clair, j'avais déjà remarqué que danielsaada était un peu comme ça mais là il y va fort.
  • Avis d'une personne de passage : c'est en effet très bizarre. Mais ceci mis à part, la question est intéressante ainsi que la piste de H.

    @Steven : peux-tu me dire de quelle vanne tu parlais dans ce fil ? Ça m'intrigue depuis quelques temps...
    Désolée aux autres de polluer la discussion avec cette intervention, mais c'est un peu difficile sans les MP...
  • Je suis troublé par cette unanimité contre moi.
    Je n'ignore pas H, j'ai estimé que son message était trop vague et guère exploitable.

    En attendant que cette hystérie retombe, voici une idée :

    prendre pour Fn les centres des boules de rayon 1/n,
    puis montrer que la partie diffuse de le mesure limite M
    n'est pas nulle (on se casserait les dents à montrer que M
    est diffuse).

    Bonne journée à tous.
  • Bon. Tu ne me réponds pas, tu remercies un autre intervenant pour son intervention, tu redemandes plusieurs fois de l'aide sans me demander de préciser ma piste... mais tu ne m'ignores pas. C'est assez original comme mode de communication ! À quel moment étais-je censé deviner que tu avais lu mon message mais que tu l'avais trouvé trop vague et guère exploitable !? Comment suis-je censé deviner ce que je dois détailler pour rendre mon message exploitable par toi ?

    J'essaye d'être plus explicite alors : Quels sont les points qui te gênent ? Est-ce la définition même de la construction ? Est-ce la manière d'établir ensuite que les limites sont diffuses ? Serais-tu satisfait, au moins dans un premier temps, par le cas métrique ? (Comme pour aléa, la topologie générale c'est un peu vieux pour moi.)
  • @ danielsaada
    Voir la Charte 4.2
  • Indication: si on montre que tout point a un voisinage qui, pour $n$ assez grand, contient au plus un des points du support de la n-ième mesure de H, alors c'est gagné grâce à JPP (Jean-Pierre Portmanteau).
  • Salut Aléa,

    C'est justement là que j'en étais. J'irai un peu plus loin : il suffit que le nombre de points dans le voisinage en question ne croisse pas trop vite. En effet si $x$ est un atome de $\mu = \lim \mu_{n_k}$ est la mesure limite d'une suite extraite et $c=\mu(\{x\})>0$, alors pour tout voisinage $V$ de $x$ on a $\liminf 2^{-n_k} |V \cap F_{n_k}| \geq c$ où $F_n$ est le support de $\mu_n$.
  • Je précise un peu ma pensée dans le cas métrique.

    1) À un couple $(x,r)$ où $x$ est un point de $K$ et $r$ un réel strictement positif, j'associe deux couples $(x',r')$ et $(x'',r'')$ de même nature où : les boules $B(x',2r')$ et $B(x'',2r'')$ sont disjointes et incluses dans la boule $B(x,r/2)$. C'est possible car aucun point n'est isolé. Je vois les deux couples $(x',r')$ et $(x'',r'')$ comme les enfants de $(x,r)$.

    2) Je pars d'un couple $(x,r)$ arbitraire et en itérant le 1) j'obtiens pour tout $n$ une famille de $2^n$ couples avec une généalogie etc.

    3) Pour tout $n$, je considère $\mu_n$ la mesure uniforme sur les $2^n$ points de la génération $n$. J'extrait une sous-suite qui converge vers une certaine mesure $\mu$.

    4) Je fixe $y$ dans $K$. Pour tout $n$, je définis un voisinage fermé $V_n$ de $y$ comme suit. Si $y$ n'est dans aucune des boules ouvertes $B(x,r)$ de la génération $n$ alors $V_n$ est le complémentaire de la réunion de ces boules. Sinon, $y$ est dans une seule de ces boules ouvertes. Je définis alors $V_n$ comme étant la boule fermée associée. Dans tous les cas, on a $\mu_k(V_n) \le 2^{-n}$ pour tout $k \ge n$. On en déduit que l'on a $\mu(V_n) \le 2^{-n}$. On en déduit alors que $\mu$ n'a pas d'atome en $y$ ce qui conclut.

    Je n'ai pas relu et il faut peut-être ajuster par endroit mais ça me semble crédible. C'est en tout cas ce que j'avais en tête en répondant à DS il y a quelques jours.
  • Salut H,

    Si ça marche je pense que ça s'adapte sans trop de problème au cas général (non-métrique). Le seul point où j'ai un doute est le point 4 : pourquoi prends-tu $V_n$ fermé ? et d'autre part dans le cas où $y$ n'est dans aucune des boules, comment s'assurer que ton $V_n$ est bien un voisinage de $y$ ?
  • Bonjour à tous,

    je crois avoir trouvé, j'envoie ma démonstration pour vérification
    et je la posterai si elle est juste.

    Merci à tous pour votre participation !
  • @Egoroff(ski).

    J'aurais effectivement dû prendre des $V_n$ ouverts, je me suis pris les pieds dans le portmanteau.

    Deuxième essai pour le 4) :

    4) Je fixe $y$ dans $K$. Pour tout $n$, je définis un voisinage ouvert $V_n$ de $y$ comme suit. Si $y$ n'est dans aucune des boules fermées $B(x,r/2)$ de la génération $n$ alors $V_n$ est le complémentaire de la réunion de ces boules. Sinon, $y$ est dans une seule des boules ouverte $B(x,r)$. Je définis alors $V_n$ comme étant la boule ouverte en question. Dans tous les cas, on a $\mu_k(V_n) \le 2^{-n}$ pour tout $k \ge n$. On en déduit que l'on a $\mu(V_n) \le 2^{-n}$. On en déduit alors que $\mu$ n'a pas d'atome en $y$ ce qui conclut.

    C'est mieux ?
  • Peut-être une autre formulation (je n'ai pas vraiment lu ce qui précède) : un espace compact parfait contient un ensemble de Cantor. De plus, l'ensemble de Cantor possède une mesure diffuse non nulle.
  • H a écrit:
    je me suis pris les pieds dans le portmanteau.
    :)-D

    Ca me paraît mieux en effet, bravo !!

    Dans le cas non-métrique, une esquisse d'adaptation de ton idée : par récurrence on définit des $F_n,\mathcal{K}_n$ où $F_n=(x_n^1,...,x_n^{2^n})$ contient les "centres" de la génération $n$, $\mathcal{K}_n=(K_n^1,...,K_n^{2^n})$ où chaque $K_n^j$ est un voisinage compact de $x_n^j$, et $\mathcal{V}_n=(V_n^1,...,V_n^{2^n})$ des ouverts de $K$, deux-à-deux disjoints, tels que $K_n^j \subset V_n^j$ (voisinages de sécurité autour des $K_n^j$). On note aussi $W_n=K \setminus \bigcup_j K_n^j$ qui est un ouvert de $K$.

    On initialise $F_1=(x_1^1)$ arbitrairement et $K_1^1=V_1^1=K$. Puis on construit la génération $n+1$ à partir de la génération $n$ en exécutant les étapes suivantes (indépendamment pour tous les $j$) :
    - on pose $x_{n+1}^{2j}=x_n^j$ ;
    - le caractère parfait de $K$ montre que l'intérieur de $K_n^j$ contient un autre point, $x_{n+1}^{2j-1} \neq x_n^j$ ;
    - comme $K$ est séparé il existe $V_{n+1}^{2j-1},V_{n+1}^{2j}$ voisinages disjoints respectifs de $x_{n+1}^{2j-1},x_{n+1}^{2j}$ ;
    - comme $K$ est compact il est localement compact et donc on peut choisir des sous-voisinages compacts $x_{n+1}^{2j-1} \in K_{n+1}^{2j-1} \subset V_{n+1}^{2j-1}$ et $x_{n+1}^{2j} \in K_{n+1}^{2j} \subset V_{n+1}^{2j}$ ;
    - par construction $W_{n+1} = W_n \cup \bigcup_{j \leq 2^n} K_n^j \setminus (K_{n+1}^{2j-1} \cup K_{n+1}^{2j})$ donc la suite $W_n$ est croissante.

    Enfin on définit $\mu_n$ comme étant la loi uniforme sur $F_n$. La compacité de $K$ et le théorème de Prokhorov assurent que la suite $(\mu_n)$ a une sous-suite convergeant faiblement vers une probabilité $\mu$.

    Etant donné un $y \in K$, on a :
    - soit $y \in W_n$ et donc pour tout $k \geq n$ : $\mu_k(W_n) \leq \mu_k(W_k) = 0$.
    - soit $y \not\in W_n$ et donc $y \in K_n^j$ pour un certain $j$ ; alors pour tout $k \geq n$, le nombre de points de $F_k$ dans $V_n^j$ est exactement $2^{k-n}$, donc $\mu_k(W_n) = 2^{-n}$.
    Dans les deux cas on a trouvé un ouvert $U$ contenant $y$ et tel que $\mu_k(U) \leq 2^{-n}$ pour tout $k \geq n$. Par conséquent la mesure limite vérifie $\mu(\{y\}) \leq \mu(U) \leq \liminf \mu_{n_k}(U) \leq 2^{-n}$, d'après le porte-manteau. Par passage à l'inf sur $n$, $y$ n'est pas un atome de $\mu$ et donc $\mu$ est diffuse.

    Ca te semble OK ?

    On attend la version de Daniel.
  • Désolé mais je ne vérifierai pas ta preuve, c'est trop coûteux pour moi de me replonger dans la locale compacité, les propriétés de séparations et compagnie ! Cela correspond en tout cas à ce que j'avais plus ou moins en tête et je n'aurais pas eu le courage de le taper aussi bien !
  • merci infiniment à tous les participants !

    j'ai trouvé ceci, hélas guère exploitable :

    http://journals.cambridge.org.sci-hub.org/action/displayAbstract?fromPage=online&aid=6965444

    Daniel
  • Merci DS pour le lien. La preuve de l'article est plus sophistiquée mais elle est en montre plus. Elle montre la densité de l'ensemble des mesures diffuses. Quelle(s) partie(s) de l'article ne comprends-tu pas ? Tu peux tout réécrire dans le cas métrique et en parlant de suites au lieu de filtres si c'est cela qui t'ennuie. Par contre la preuve est marquée "analyse fonctionnelle". On travaille dans l'espace des mesures de probabilité sur le compact en question. Ça c'est vraiment l'esprit de la preuve et on ne peut pas y couper.
  • Bonjour H,

    comment as_tu fait pour lire l'article ?
    je publierai ma preuve demain.
    bonne journée
    Daniel
  • Aucune idée ! Ce matin, de chez-moi et sans aucun abonnement j'y ai eu accès. Je vois que ce n'est maintenant plus le cas. C'est assez mystérieux.

    Le plan était le suivant, si ma mémoire est bonne.

    1) On considère l'espace $E$ des mesures de probabilité sur le compact considéré, muni de la convergence usuelle pour ce genre de mesures (convergence contre les applications continues).

    2) Pour tout $\eta > 0 $ on considère $C(\eta)$ le sous-ensemble constitué des mesures contenant au moins un atome de masse au moins $\eta$. C'est un fermé (c'est une conséquence du fameux porte-manteau).

    3) S'il n'existait pas de mesure diffuse, alors $E$ serait la réunion des $C(n^{-1})$. Par Baire, l'un des $C(n^{-1})$ serait alors d'intérieur non vide.

    4) Or les $C(\eta)$ sont d'intérieur vide (car on peut décomposer un atome en atomes plus petits).

    Il utilisait quelque part la densité des mesures purement atomiques, peut-être pour montrer le 4, mais je ne vois plus l'intérêt.
  • Tiens ben l'article est de nouveau accessible librement, apparement ça oscille ! Ou alors il y a une politique complexe.

    Après une rapide relecture j'ai l'impression que la densité des mesures purement atomique est effectivement inutile.

    Le plan est donc le suivant : les $C(n^{-1})$ sont des fermés d'intérieur vide ; leur réunion est donc d'intérieur vide (Baire) ; or leur réunion est précisémment l'ensemble des mesures diffuses.
  • Joli... L'application de Baire utilise la compacité de $E$, qui vient de la compacité de l'espace, et le caractère parfait sert à montrer le 4 c'est ça ?
  • Oui.
  • Merci, du coup on voit bien où servent les deux hypothèses.
  • Oui, mais c'était déjà clair pour l'autre preuve (et le rôle était essentiellement le même) :

    A) La compacité pour dire que l'ensemble des valeurs d'adhérences d'une suite est non vide / la compacité pour appliquer Baire et dire qu'un certain ensemble est non vide.

    B) L'absence de points isolés pour construire une suite de mesures convenables où les poids des atomes tendent vers $0$ / l'absence de points isolés pour dire qu'on est arbitraiement proche d'une mesure avec des atomes de poids plus petit.
  • Bonsoir à tous,

    comme promis, voici notre solution

    ne pas tenir compte de la rédaction précipitée.

    Daniel
  • H : Tu as raison, d'ailleurs on exhibe une suite de mesures telles que $\mu_n \in C(2^{-n})$. Mais du coup il me semble que "mon" adaptation de ta preuve dans le cas non métrique n'est pas optimale, au sens où elle utilise deux fois la compacité (une fois pour l'existence d'une valeur d'adhérence, et une fois dans la construction des voisinages de sécurité).

    Daniel : OK merci d'avoir pris le temps de taper tout ça (même si ça aurait été mieux en LaTeX :) ). C'est essentiellement la même idée que la démonstration de H, avec un point de vue "fonctionnel". Attention dans ton cas 1 : $f_n$ est "une" (pas "la") fonction qui vaut etc. et de plus il faut imposer $f_n \leq 1$ pour pouvoir majorer $\int f_n dM_m$.
Connectez-vous ou Inscrivez-vous pour répondre.
Success message!